Eyes/Ears/Sensory

Ace your homework & exams now with Quizwiz!

13. The nurse is assessing a 48-year-old patient for presbyopia. Which equipment will the nurse need to obtain before the examination? a. Penlight b. Tono-pen c. Jaeger chart d. Snellen chart

ANS: C Presbyopia is the normal loss of near vision that occurs with age and is assessed using a Jaeger chart. The Snellen chart, penlight, and the Tono-pen are used when assessing for other visual disorders.

10. In order to assess the visual acuity for a patient in the outpatient clinic, the nurse will need to obtain a (an) a penlight. B Amsler grid. C Snellen chart. D ophthalmoscope.

ANS: C The Snellen chart is used to check visual acuity. An ophthalmoscope, penlight, and Amsler grid also may be used during an eye examination, but they are not helpful in assessing visual acuity.

7. A patient is diagnosed with adult inclusion conjunctivitis (AIC) caused by Chlamydia trachomatis. Which of these actions will be included in the plan of care? a. Discussing the need for sexually transmitted disease testing b. Applying topical corticosteroids to prevent further inflammation c. Assisting with applying for community visual rehabilitation services d. Educating about the use of antiviral eyedrops to treat the infection

ANS: A Patients with AIC have a high risk for concurrent genital Chlamydia infection and should be referred for STD testing. AIC is treated with antibiotics; antiviral and corticosteroid medications are not appropriate therapies. Although some types of Chlamydia infection do cause blindness, AIC does not lead to blindness, so referral for visual rehabilitation is not appropriate.

20. A patient with Ménière's disease is admitted with vertigo, nausea, and vomiting. Which nursing intervention will be included in the care plan? a. Keep the patient's room darkened. b. Encourage oral fluids to 3000 ml daily. c. Change the patient's position every 2 hours. d. Keep the head of the bed elevated 30 degrees.

ANS: A A darkened, quiet room will decrease the symptoms of the acute attack of Ménière's disease. Since the patient will be nauseated during an acute attack, fluids are administered intravenously. Position changes will cause vertigo and nausea. The head of the bed can be positioned for patient comfort.

39. The priority nursing diagnosis for a patient with Ménière's disease who is experiencing an acute attack is a. risk for falls related to dizziness. b. impaired verbal communication related to tinnitus. c. self-care deficit (bathing and dressing) related to vertigo. d. imbalanced nutrition: less than body requirements related to nausea.

ANS: A All the nursing diagnoses are appropriate, but because sudden attacks of vertigo can lead to "drop attacks," the major focus of nursing care is to prevent injuries associated with dizziness.

36. Which of these nursing activities is appropriate for the RN working in the eye clinic to delegate to experienced nursing assistive personnel (NAP)? a. Application of a warm compress to a patient's hordeolum b. Assessment of a patient with possible bacterial conjunctivitis c. Instruction about hand washing for a patient with herpes keratitis d. Administration of antiviral drops to a patient with a corneal ulcer

ANS: A Application of cold and warm packs is included in NAP education and the ability to accomplish this safely would be expected for a nursing assistant working in an eye clinic. Medication administration, patient teaching, and assessment are high-level skills appropriate for the education and legal practice level of the RN.

28. A patient is scheduled for a right cataract extraction and intraocular lens implantation at an ambulatory surgical center in 2 weeks. During the preoperative assessment of the patient in the physician's office, it is most important for the nurse to assess a. the visual acuity of the patient's left eye. b. for a white pupil in the patient's right eye. c. how long that the patient has had the cataract. d. for a history of reactions to general anesthetics.

ANS: A Because it can take several weeks before the maximum improvement in vision occurs in the right eye, patient safety and independence are determined by the vision in the left eye. Cataract surgery is done using local anesthetics rather than general anesthetics. A white pupil in the operative eye would not be unusual for a patient scheduled for cataract removal and lens implantation. The length of time that the patient has had the cataract will not impact on the perioperative care.

24. A patient with hearing loss asks the nurse about the use of a cochlear implant. Which information will the nurse include when replying to the patient? a. Cochlear implants require training in order to receive the full benefit. b. Cochlear implants are not useful for patients with congenital deafness. c. Cochlear implants are most helpful as an early intervention for presbycusis. d. Cochlear implants improve hearing in patients with conductive hearing loss.

ANS: A Extensive rehabilitation is required after cochlear implants in order for patients to receive the maximum benefit. Hearing aids, rather than cochlear implants, are used initially for presbycusis. Cochlear implants are used for sensorineural hearing loss and would not be helpful for conductive loss. They are appropriate for some patients with congenital deafness

23. An older adult patient with presbycusis is fitted with binaural hearing aids. Which information will the nurse include when teaching the patient how to use the hearing aids? a. Experiment with volume and hearing ability in a quiet environment initially. b. Keep the volume low on the hearing aids for the first week while adjusting to them. c. Add the second hearing aid after making the initial adjustment to the first hearing aid. d. Wear the hearing aids for about an hour a day at first, gradually increasing the time of use.

ANS: A Initially the patient should use the hearing aids in a quiet environment like the home, experimenting with increasing and decreasing the volume as needed. There is no need to gradually increase the time of wear. The patient should experiment with the level of volume to find what works well in various situations. Both hearing aids should be used.

33. A patient who had cataract extraction and intraocular lens implantation the previous day calls the eye clinic and gives the nurse all of the following information. Which information is the priority to communicate to the health care provider? a. The patient has eye pain rated at a 5 (on a 0-10 scale). b. The patient has questions about the ordered eyedrops. c. The patient has poor depth perception when wearing an eye patch. d. The patient complains that the vision has "not improved very much."

ANS: A Postoperative cataract surgery patients usually experience little or no pain, so pain at a 5 on a 10-point pain level may indicate complications such as hemorrhage, infection, or increased intraocular pressure. The other information given by the patient indicates a need for patient teaching but does not indicate that complications of the surgery may be occurring

19. The nurse is assessing a patient who has recently been treated with amoxicillin (Amoxil) for acute otitis media of the right ear. Which assessment data obtained by the nurse is of most concern? a. The patient has a temperature of 100.6° F. b. The patient complains of "popping" in the ear. c. The patient frequently asks the nurse to repeat information. d. The patient states that the right ear has a feeling of fullness.

ANS: A The fever indicates that the infection may not be resolved and the patient might need further antibiotic therapy. A feeling of fullness, "popping" of the ear, and decreased hearing are symptoms of otitis media with effusion. These symptoms are normal for weeks to months after an episode of acute otitis media and usually resolve without treatment.

11. A patient being admitted to the hospital has an eye patch in place and tells the nurse "I had a recent eye injury, so I need to wear this patch for a few weeks." Which nursing diagnosis will the nurse include in the plan of care? a. Risk for falls related to current decrease in stereoscopic vision b. Ineffective health maintenance related to inability to see surroundings c. Disturbed body image related to eye trauma and need to wear eye patch d. Ineffective denial related to inability to admit the impact of the eye injury

ANS: A The loss of stereoscopic vision created by the eye patch impairs the patient's ability to see in three dimensions and to judge distances. It also increases the risk for falls. There is no evidence in the assessment data for ineffective denial, disturbed body image, or ineffective health maintenance.

12. A patient in the eye clinic is scheduled for refractometry. Which information will the nurse include in patient teaching? a. "You will need to wear sunglasses for a few hours after the exam." b. "The surface of your eye will be numb while the doctor does the exam." c. "You should not take any of your eye medicines before the examination." d. "The doctor will shine a bright light into your eye during the examination."

ANS: A The pupil is dilated by using cycloplegic medications during refractometry. This effect will last several hours and cause photophobia. The other teaching would not be appropriate for a patient who was having refractometry.

4. The nurse at the eye clinic advises all patients to wear sunglasses that protect the eyes from ultraviolet light because ultraviolet sunlight exposure is associated with the development of a. cataracts. b. glaucoma. c. anisocoria. d. exophthalmos.

ANS: A Ultraviolet light exposure is associated with the accelerated development of cataracts. Glaucoma is caused by increased intraocular pressure, exophthalmos is associated with hyperthyroidism, and anisocoria can occur normally in a small percentage of the population or may be caused by injury or central nervous system disorders.

30. Which action should the RN who is working in the eye and ear clinic delegate to an LPN/LVN? a. Use a Snellen chart to check a patient's visual acuity. b. Evaluate a patient's ability to insert soft contact lenses. c. Teach a patient with otosclerosis about use of sodium fluoride and vitamin D. d. Assess the external auditory canal for signs of irritation caused by a hearing aid.

ANS: A Using standardized screening tests such as a Snellen chart to test visual acuity is included in LPN education and scope of practice. Evaluation, assessment, and patient education are higher level skills that require RN education and scope of practice.

5. Assessment of a patient's visual acuity reveals that the left eye can see at 20 feet what a person with normal vision can see at 40 feet and the right eye can see at 20 feet what a person with normal vision can see at 50 feet. The nurse records which of the following findings as visual acuity? a. OS 20/40; OD 20/50 b. OU 20/40; OS 50/20 c. OD 20/40; OS 20/50 d. OU 40/20; OD 50/20

ANS: A When documenting visual acuity, the first number indicates the standard (for normal vision) of 20 feet and the second number indicates the line that the patient is able to read when standing 20 feet from the Snellen chart. OS is the abbreviation for left eye and OD is the abbreviation for right eye. The remaining three answers do not correctly describe the patient's visual acuity.

8. Which topic will the nurse include in patient teaching after a patient has had outpatient cataract surgery and lens implantation? a. Use of oral opioids for pain control b. Administration of antibiotic eyedrops c. Importance of coughing and deep breathing exercises d. Need for bed rest for the first 24 hours after the surgery

ANS: B Antibiotic and corticosteroid eyedrops are commonly prescribed after cataract surgery, and the patient should be able to administer them using safe technique. Pain is not expected after cataract surgery and opioids will not be needed. Coughing and deep breathing exercises are not needed since a general anesthetic agent is not used. There is no bed rest restriction after cataract surgery.

4. A patient is seen at a clinic for repeated hordeolum of the eyes during the last 6 months. To help prevent further infection, the nurse advises the patient to a. apply cold compresses at the first sign of recurrence. b. discard all open or used cosmetics used near the eyes. c. wash the scalp and eyebrows with an antiseborrheic shampoo. d. be evaluated for the presence of sexually transmitted diseases (STDs).

ANS: B Hordeolum (styes) are commonly caused by Staphylococcus aureus, which may be present in cosmetics that the patient is using. Warm compresses are recommended to treat hordeolum. Antiseborrheic shampoos are recommended for seborrheic blepharitis. Patients with adult inclusion conjunctivitis, which is caused by Chlamydia trachomatis, should be referred for STD testing.

5. Which topic will the nurse plan to include when teaching the patient with herpes simplex keratitis of the left eye about management of the infection? a. How to apply an occlusive dressing to the affected eye b. Need for frequent hand washing and avoiding touching the eyes c. Application of antibiotic drops to the left eye several times daily d. Use of corticosteroid ophthalmic ointment to decrease inflammation

ANS: B The best way to avoid the spread of infection from one eye to another is to avoid rubbing or touching the eyes and to use careful hand washing when touching the eyes is unavoidable. Occlusive dressings are not used for herpes keratitis. Herpes simplex is a virus and antibiotic drops will not be prescribed. Topical corticosteroids typically are not ordered because they can contribute to a longer course of infection and more complications.

To evaluate the effectiveness of the prescribed bifocals for a patient with myopia and presbyopia, the nurse in the eye clinic will check the patient for a. strength of the eye muscles. b. both near and distant vision. c. cloudiness in the eye lenses. d. intraocular pressure changes.

ANS: B The lenses are prescribed to correct the patient's near and distant vision. The nurse also may assess for cloudiness of the lenses, increased intraocular pressure, and eye movement, but these data will not evaluate whether the patient's bifocals are effective

21. Which observation by the nurse when examining a patient's auditory canal and tympanic membrane is a priority to report to the health care provider? a. There is a cone of light visible. b. The tympanum is bluish-tinged. c. Cerumen is present in the auditory canal. d. The skin in the ear canal is dry and scaly.

ANS: B A bluish-tinged tympanum ca cur with acute otitis media, which requires immediate care to prevent perforation of the tympanum. Cerumen in the ear canal may need to be removed before proceeding with the examination but is not unusual or pathologic. The presence of a cone of light on the eardrum is normal. Dry and scaly skin in the ear canal may need further assessment but does not require urgent care.

40. Which information about a patient who had a stapedotomy yesterday is most important for the nurse to communicate to the health care provider? a. The patient complains of "congestion" in the ear. b. The patient's oral temperature is 100.6° F (38.1° C). c. The patient says "My hearing is worse now than it was right after surgery." d. There is a small amount of dried bloody drainage on the patient's dressing.

ANS: B An elevated temperature may indicate a postoperative infection. Although the nurse would report all the data, a temporary decrease in hearing, bloody drainage on the dressing, and a feeling of congestion because of the accumulation of blood and drainage in the ear are common after this surgery.

35. The charge nurse observes a newly hired nurse performing all of the following interventions for a patient who has just arrived in the postanesthesia care unit after having right cataract removal and an intraocular lens implant. Which one requires that the charge nurse intervene? a. The nurse leaves the eye shield in place. b. The nurse encourages the patient to cough. c. The nurse elevates the patient's head to 45 degrees. d. The nurse applies corticosteroid drops to the right eye.

ANS: B Because coughing will increase intraocular pressure, patients are generally taught to avoid coughing during the acute postoperative time. The other actions are appropriate for a patient after having this surgery.

1. Which information will the nurse include when teaching a patient about routine glaucoma testing? a. The test involves reading a Snellen chart at a distance of 20 feet. b. Application of a Tono-pen to the surface of the eye will be needed. c. The examination includes checking the pupil's reaction to a bright light. d. Medications to dilate the pupil will be used before testing for glaucoma.

ANS: B Glaucoma is caused by an increase in intraocular pressure, which would be measured using the Tono-pen. The other techniques are used in testing for other eye disorders

27. The nurse at the outpatient surgery unit obtains all of this information about a patient who is scheduled for cataract extraction and implantation of an intraocular lens. Which information has the most immediate implications for the patient's care? a. The patient has not eaten anything for 8 hours. b. The patient takes three antihypertensive medications. c. The patient gets nauseated with general anesthesia. d. The patient has had blurred vision for several years.

ANS: B Mydriatic medications used for pupil dilation are sympathetic nervous system stimulants and may increase heart rate and blood pressure. Using punctal occlusion when administering the mydriatic and monitoring of blood pressure are indicated for this patient. Patients are expected to be NPO for 6 to 8 hours before the surgical procedure. Blurred vision is an expected finding with cataracts. Cataract extraction and intraocular lens implantation are done using local anesthesia.

12. To determine whether treatment is effective for a patient with primary open-angle glaucoma (POAG), the nurse will evaluate the patient for improvement in a. eye pain. b. visual field. c. blurred vision. d. depth perception.

ANS: B POAG develops slowly and without symptoms except for a gradual loss of visual field. Acute closed-angle glaucoma may present with excruciating pain, colored halos, and blurred vision. Problems with depth perception are not associated with POAG.

22. When the nurse is admitting a 78-year-old patient, the patient repeatedly asks the nurse to "speak up so that I can hear you." Which action should the nurse take? a. Overenunciate while speaking. b. Speak normally but more slowly. c. Increase the volume when speaking. d. Use more facial expressions when talking.

ANS: B Patient understanding of the nurse's speech will be enhanced by speaking at a normal tone, but more slowly. Increasing the volume, overenunciating, and exaggerating facial expressions will not improve the patient's ability to comprehend the nurse.

2. Which assessment information obtained by the nurse when performing an eye examination for a 78-year-old patient indicates that more extensive examination of the eyes is needed? a. The patient's sclerae are light yellow in color. b. The patient complains of persistent photophobia. c. The pupil recovers slowly after being stimulated by a penlight. d. There is a whitish gray ring encircling the periphery of the iris.

ANS: B Photophobia is not a normally occurring change with aging and would require further assessment. The other assessment data are common gerontologic differences and would not be unusual in a 78-year-old patient.

7. The nurse is observing a student who is preparing to perform an ear examination of a 24-year-old patient. The nurse will need to intervene if the student a chooses a speculum smaller than the ear canal. b pulls the auricle of the ear down and backward. c stabilizes the hand holding the otoscope on the patient's head. D stops inserting the otoscope after observing impacted cerumen.

ANS: B The auricle should be pulled up and back when assessing an adult. The other actions are appropriate when performing an ear examination.

15. During the nursing history, a patient complains of dizziness when bending over and of nausea and dizziness associated with physical activities. The nurse will plan to teach the patient about a. tympanometry. b. rotary chair testing. c. pure-tone audiometry. d. bone-conduction testing.

ANS: B The patient's clinical manifestations of dizziness and nausea suggest a disorder of the labyrinth, which controls balance and contains three semicircular canals and the vestibule. Rotary chair testing is used to test vestibular function. The other tests are used to test for problems with hearing.

15. A patient who has bacterial endophthalmitis in the left eye is restless, frequently asking whether the eye is healing and whether removal of the eye will be necessary. Based on the assessment data, which nursing diagnosis is appropriate? a. Grieving related to current loss of functional vision b. Anxiety related to the possibility of permanent vision loss c. Situational low self-esteem related to loss of visual function d. Risk for falls related to inability to see environmental hazards

ANS: B The patient's restlessness and questioning of the nurse indicate anxiety about the future possible loss of vision. Because the patient can see with the right eye, functional vision is relatively intact and the patient is not at a high risk for falls. There is no indication of impaired self-esteem at this time.

6. Which nursing action is included when assessing a patient's visual field? a. Position the patient 20 feet from the Snellen chart. b. Have the patient cover one eye while facing the nurse. c. Instruct the patient to follow a moving object using only the eyes. d. Shine a light into one pupil and observe the response for both pupils.

ANS: B To perform confrontation visual field testing, the patient faces the examiner and covers one eye, then counts the number of fingers that the examiner brings into the visual field. The other actions are needed to test for visual acuity, extraocular movements, and consensual pupil response.

32. The nurse notes that nursing assistive personnel (NAP) perform all the following actions when caring for a patient with Ménière's disease who is experiencing an acute attack. Which action by NAP indicates that the nurse should intervene immediately? a. NAP raise the side rails on the bed. b. NAP turn on the patient's television. c. NAP turn the patient to the right side. d. NAP place an emesis basin at the bedside.

ANS: B Watching television may exacerbate the symptoms of an acute attack of Ménière's disease. The other actions are appropriate.

2. A patient is seen in the ophthalmology clinic and diagnosed with recurrent staphylococcal and seborrheic blepharitis. The nurse will plan to teach the patient about a. saline irrigation of the eyes. b. surgical removal of the lesion. c. using baby shampoo to clean the lids. d. the use of cool compresses to the eyes.

ANS: C Baby shampoo is used to soften and remove crusts associated with blepharitis. The other interventions are not used in treating this disorder.

3. When performing an eye examination, the nurse will assess for accommodation by a. covering one eye for 1 minute and noting the pupil reaction when the cover is removed. b. shining a light into the patient's eye and watching the pupil response in the opposite eye. c. observing the pupils when the patient focuses on a close object and then on a distant object. d. touching the patient's pupil with a small piece of sterile cotton and watching for a blink reaction.

ANS: C Accommodation is defined as the ability of the lens to adjust to various distances. The other nursing actions also may be part of the eye examination, but they do not test for accommodation.

21. The home health nurse observes a patient taking these actions when self-administering eardrops. Which patient action indicates a need for more teaching? a. The patient leaves the ear wick in place while administering the drops. b. The patient lies down before and for 2 minutes after administering the drops. c. The patient gets the eardrops out of the refrigerator just before administering the drops. d. The patient holds the tip of the dropper 1 cm above the ear while administering the drops.

ANS: C Administration of cold eardrops can cause dizziness because of stimulation of the semicircular canals. The other patient actions are appropriate.

18. A patient with chronic otitis media is scheduled for a tympanoplasty. Before surgery, the nurse teaches the patient that postoperative expectations include a. keeping the head elevated. b. the need for prolonged bed rest. c. avoidance of coughing or blowing the nose. d. continuous antibiotic irrigation of the ear canal.

ANS: C Coughing or blowing the nose increases pressure in the eustachian tube and middle ear cavity and disrupts postoperative healing. There is no postoperative need for prolonged bed rest, elevation of the head, or continuous antibiotic irrigation.

31. The camp nurse is caring for a patient who is complaining of bilateral eye pain after a campfire log exploded, sending sparks into the patient's eyes. Which of these actions will the nurse take first? a. Apply ice packs to the eyes. b. Flush the eyes with sterile saline. c. Cover the eyes with dry sterile patches and protective eye shields. d. Apply antiseptic ophthalmic ointment from the first aid kit to the eyes.

ANS: C Emergency treatment of a burn or foreign-body injury to the eyes includes protecting the eyes from further injury by covering them with dry sterile dressings and protective shields. Flushing of the eyes immediately is indicated only for chemical exposure. Except in the case of chemical exposure, the nurse should not begin treatment until the patient has been assessed by a health care provider and orders are available.

17. A patient with external otitis has an ear wick placed and a new prescription for antibiotic otic drops. After the nurse provides patient teaching, which patient statement indicates that more instruction is needed? a. "I may use aspirin or acetaminophen (Tylenol) for pain relief." b. "I should apply the eardrops to the cotton wick in my ear canal." c. "I should clean my ear canal daily with a cotton-tipped applicator." d. "I may use warm compresses to the outside of my ear for comfort."

ANS: C Insertion of instruments such as cotton-tipped applicators into the ear should be avoided. The other patient statements indicate that the teaching has been successful.

9. The nurse is obtaining a health history for a 64-year-old patient with glaucoma who is a new patient at the eye clinic. Which information given by the patient will have the most implications for the patient's treatment? A "I use aspirin when I have a sinus headache." B "I have had frequent episodes of conjunctivitis." C "I take metoprolol (Lopressor) daily for angina." D "I have not had an eye examination for 10 years."

ANS: C It is important to note whether the patient takes any β-adrenergic blockers because this category of medications also is used to treat glaucoma, and there may be an increase in adverse effects. The use of aspirin does not increase intraocular pressure and is safe for patients with glaucoma. Although older patients should have yearly eye examinations, the treatment for this patient will not be affected by the 10-year gap in eye care. Conjunctivitis does not increase the risk for glaucoma.

29. When admitting a patient for surgery, the nurse learns that the patient has functional blindness and that the spouse has cared for the patient for many years. During the initial assessment of the patient, it is most important for the nurse to a. obtain more information about the cause of the patient's vision loss. b. obtain information from the spouse about the patient's special needs. c. make eye contact with the patient and ask about any need for assistance. d. perform an evaluation of the patient's visual acuity using a Snellen chart.

ANS: C Making eye contact with a partially sighted patient allows the patient to hear the nurse more easily and allows the nurse to assess the patient's facial expressions. The patient (rather than the spouse) should be asked about any need for assistance. The information about the cause of the vision loss and assessment of the patient's visual acuity are not priorities during the initial assessment.

13. A patient with glaucoma who has been using timolol (Timoptic) drops for several days tells the nurse that the eyedrops cause eye burning and visual blurriness for a short time after administration. The best response to the patient's statement is a. "These are normal side effects of the drug, which should become less noticeable with time." b. "If you occlude the puncta after you administer the drops, it will help relieve these side effects." c. "The drops are uncomfortable, but it is very important for you to use them as prescribed to retain your vision." d. "These symptoms are caused by glaucoma and may indicate a need for an increased dosage of the eyedrops."

ANS: C Patients should be instructed that eye discomfort and visual blurring are expected side effects of the ophthalmic drops but that the drops must be used to prevent further visual-field loss. The temporary burning and visual blurriness might not lessen with ongoing use, are not relieved by avoiding systemic absorption, and are not symptoms of glaucoma.

16. When the nurse is taking a health history of a new patient at the ear clinic, the patient states, "I always sleep with the radio on." Which follow-up question is most appropriate to obtain more information about possible hearing problems? a. "Do you grind your teeth at night?" b. "What time do you usually fall asleep?" c. "Have you noticed any ringing in your ears?" d. "Are you ever dizzy when you are lying down?"

ANS: C Patients with tinnitus may use masking techniques, such as playing a radio, to block out the ringing in the ears. The responses "Do you grind your teeth at night?" and "Have you noticed any ringing in your ears?" would be used to obtain information about other ear problems, such as vestibular disorders and referred temporomandibular joint (TMJ) pain. The response "What time do you usually fall asleep?" would not be helpful in assessing problems with the patient's ears.

37. A patient with a head injury after a motor vehicle accident arrives in the emergency department (ED) complaining of shortness of breath and severe eye pain. Which action will the nurse take first? a. Elevate the head to 45 degrees. b. Administer the ordered analgesic. c. Check the patient's oxygen saturation. d. Examine the eye for evidence of trauma.

ANS: C The priority action for a patient after a head injury is to assess and maintain airway and breathing. Because the patient is complaining of shortness of breath, it is essential that the nurse assess the oxygen saturation. The other actions also are appropriate but are not the first action the nurse will take.

9. In reviewing a 50-year-old patient's medical record, the nurse notes that the last eye examination revealed an intraocular pressure of 28 mm Hg. The nurse will plan to assess a. visual acuity. b. pupil reaction. c. color perception. d. peripheral vision.

ANS: D The patient's increased intraocular pressure indicates glaucoma, which decreases peripheral vision. Because central visual acuity is unchanged by glaucoma, assessment of visual acuity could be normal even if the patient has worsening glaucoma. Color perception and pupil reaction to light are not affected by glaucoma.

6. A new patient with 20/200 vision (with the use of corrective lenses) is being cared for by the nurse in the eye clinic. The nurse will plan to teach the patient about a. how to use a cane safely. b. how to access audio books. c. where Braille instruction is available. d. where to obtain specialized magnifiers.

ANS: D Various types of magnifiers can enhance the remaining vision enough to allow the performance of many tasks and activities of daily living (ADLs). Audio books, Braille instruction, and canes usually are reserved for patients with no functional vision.

3. Which action should the nurse take when assisting a totally blind patient to walk to the bathroom? a. Take the patient by the arm and lead the patient slowly to the bathroom. b. Have the patient place a hand on the nurse's shoulder and guide the patient. c. Stay beside the patient and describe any obstacles on the path to the bathroom. d. Walk slightly ahead of the patient and allow the patient to hold the nurse's elbow.

ANS: D When using the sighted-guide technique, the nurse walks slightly in front and to the side of the patient and has the patient hold the nurse's elbow. The other techniques are not as safe in assisting a blind patient.

20. The nurse in the eye clinic is examining a 65-year-old patient who says "I see small spots that move around in front of my eyes." Which action will the nurse take first? a. Immediately have the ophthalmologist evaluate the patient. b. Explain that spots and "floaters" are a normal part of aging. c. Inform the patient that these spots may indicate damage to the retina. d. Use an ophthalmoscope to examine the posterior chamber of the eyes.

ANS: D Although "floaters" are usually caused by vitreous liquefaction and are common in aging patients, they can be caused by hemorrhage into the vitreous humor or by retinal tears, so the nurse's first action will be to examine the retina and posterior chamber. Although the ophthalmologist will examine the patient, the presence of spots or floaters in a 65-year-old is not an emergency. The spots may indicate retinal damage, but the nurse should assess the eye further before discussing this with the patient.

14. A patient arrives in the emergency department complaining of eye itching and pain caused by sleeping with contact lenses in place. To facilitate further examination of the eye, the nurse will anticipate the need for a. a tonometer. b. eye patching. c. a refractometer. d. fluorescein dye.

ANS: D Eye itching and pain suggest a possible corneal abrasion or ulcer, which can be visualized using fluorescein dye. The other items listed would not be helpful in determining the cause of this patient's symptoms.

25. Which teaching will the nurse implement for a patient who has just been diagnosed with viral conjunctivitis? a. Explain the purpose of antiviral eyedrops. b. Show how to perform eye irrigation safely. c. Instruct about how to insert soft contact lenses. d. Demonstrate appropriate hand-washing technique.

ANS: D Hand washing is the major means to prevent the spread of conjunctivitis. Antiviral drops and eye irrigation will not be helpful in shortening the disease process. Contact lenses should not be used when patients have conjunctivitis because they can further irritate the conjunctiva.

18. When taking a health history from a new patient in the outpatient clinic, which information may indicate the need to perform a focused hearing assessment? a. The patient uses albuterol (Proventil) for acute asthma. b. The patient takes atenolol (Tenormin) to prevent angina. c. The patient uses acetaminophen (Tylenol) frequently for headaches. d. The patient has taken ibuprofen (Advil) for 20 years to treat arthritis.

ANS: D Nonsteroidal anti-inflammatory drugs (NSAIDs) are potentially ototoxic. Acetaminophen, atenolol, and albuterol are not associated with hearing loss.

17. Which finding by the nurse during the admission assessment for a patient may indicate that the patient is at risk for falls while hospitalized? a. Lateralization with Weber test b. Positive result for Rinne testing c. Inability to hear a low-pitched whisper d. Nystagmus when head is turned rapidly

ANS: D Nystagmus suggests that the patient may have problems with balance related to disease of the vestibular system. The other tests are used to check hearing; abnormal results for these do not indicate potential problems with balance.

26. Which information will the nurse include when teaching a patient with keratitis caused by herpes simplex type 1? a. Application of corticosteroid ophthalmic ointment to the eyes. b. Application of povidone-iodine (Betadine) gel around the eye. c. Avoidance of nonsteroidal anti-inflammatory drugs (NSAIDs). d. Importance of taking all of the ordered oral acyclovir (Zovirax).

ANS: D Oral acyclovir may be ordered for herpes simplex infections. Corticosteroid ointments are usually contraindicated because they prolong the course of the infection. Although Betadine gel may be applied to the skin around the eyes for herpes zoster (varicella) infections, it is not used for herpes simplex infections. NSAIDs can be used to treat the pain associated with keratitis.

14. A patient who is being admitted to the hospital for abdominal pain and nausea tells the nurse about a history of glaucoma. Which of these prescribed medications should the nurse question? a. morphine sulfate 4 mg IV b. diazepam (Valium) 5 mg IV c. betaxolol (Betoptic) 0.25% eyedrops d. scopolamine patch (Transderm Scop) 1.5 mg

ANS: D Scopolamine is a parasympathetic blocker and will relax the iris, causing blockage of aqueous humor outflow and an increase in intraocular pressure. The other medications are appropriate for this patient.

8. When obtaining a health history from a 52-year-old patient, which patient statement is most important to communicate to the health care provider? A "My vision seems blurry now when I read." B "I have noticed that my eyes are drier now." C "It is hard for me to see when I drive at night." D "The peripheral part of my vision is decreased."

ANS: D The decrease in peripheral vision may indicate glaucoma, which is not a normal visual change associated with aging and requires rapid treatment. The other patient statements indicate visual problems (presbyopia, dryness, and lens opacity) that are considered a normal part of aging.

38. These medications are prescribed by the health care provider for a patient who has just been admitted to a hospital with acute angle-closure glaucoma. Which medication should the nurse give first? a. morphine sulfate 4 mg intravenously b. betaxolol (Betoptic) 1 drop in each eye c. acetazolamide (Diamox) 250 mg orally d. mannitol (Osmitrol) 100 mg intravenously

ANS: D The most immediate concern for the patient is to lower intraocular pressure, which will occur most rapidly with IV administration of a hyperosmolar diuretic such as mannitol. The other medications also are appropriate for a patient with glaucoma but would not be the first medication administered

34. Which assessment finding in a patient who was struck in the right eye with a baseball is a priority for the nurse to communicate to the health care provider in the emergency department? a. The patient complains of a right-sided headache. b. The sclerae on the right eye have broken blood vessels. c. The area around the right eye is bruised and tender to the touch. d. The patient complains of "a curtain" blocking part of the visual field.

ANS: D The patient's sensation that a curtain is coming across the field of vision suggests retinal detachment and the need for rapid action to prevent blindness. The other findings would be expected with the patient's history of being hit in the eye with a ball

11. A patient with age-related macular degeneration has just had photodynamic therapy. Which statement by the patient indicates that the discharge teaching has been effective? a. "I will need to use bright lights to read for at least the next week." b. "I will use drops to keep my pupils dilated until my appointment." c. "I will not use facial lotions near my eyes during the recovery period." d. "I will keep covered with long-sleeved shirts and pants for the next 5 days."

ANS: D The photosensitizing drug used for photodynamic therapy is activated by exposure to bright light and can cause burns in areas exposed to light for 5 days after the treatment. There are no restrictions on use of facial lotions, medications to keep the pupils dilated would not be appropriate, and bright lights would increase the risk for damage caused by the treatment

19. Which action will the nurse include in the plan of care for a patient who has vestibular disease? a. Check Rinne and Weber tests. b. Face the patient when speaking. c. Enunciate clearly when speaking. d. Monitor the patient's ability to ambulate safely.

ANS: D Vestibular disease affects balance so the nurse should monitor the patient during activities that require balance. The other action might be used for patients with hearing disorders.

16. To decrease the risk for future hearing loss, which action should the nurse working with college students at the on-campus health clinic implement? a. Arrange to include otoscopic examinations for all patients. b. Administer rubella immunizations to all students at the clinic. c. Discuss the importance of limiting exposure to very amplified music. d. Teach patients to regularly irrigate the ear to decrease cerumen impaction.

ANS:C The nurse should discuss the impact of amplified music on hearing with young adults and discourage listening to very amplified music, especially for prolonged periods. Cerumen may need to be regularly removed for older patients, but this is not a routine need for younger adults. Only women of childbearing age who have not been previously vaccinated or exposed to rubella will require immunization. Otoscopic examinations are not necessary for all patients

10. A patient with a left retinal detachment has a pneumatic retinopexy procedure. Which information will be included in the discharge teaching plan? a. The use of bilateral eye patches to reduce movement of the operative eye b. The need to wear dark or tinted glasses to protect the eyes from bright light c. The procedure for sterile dressing changes when the eye dressing is saturated d. The purpose of maintaining the head in a prescribed position for several weeks

ANS:D Following pneumatic retinopexy, the patient will need to position the head so the air bubble remains in contact with the retinal tear. The dark lenses and bilateral eye patches are not required after this procedure. Saturation of any eye dressings would not be expected following this procedure.

The nurse is working in a long-term care facility. Which clues does the nurse note which suggests that the client is not hearing what the nurse said? Select all that apply. A) The client does not want to be social. B) The client responds inappropriately. C) The client asks for surrounding sounds be increased. D) The client nods the head and smiles. E) The client withdraws from activity.

An A, B, D, E Feedback: A nurse notices clues that the client is having difficulty hearing. Clues include not wanting to be social because interaction is difficult, responding/answering questions inappropriately, nodding and smiling no matter what the answer is, and withdrawing from activity to name a few. Increasing the surrounding sounds makes it more difficult to hear.

3. Which of the following actions should be taken when there is an insect in the ear? A) Instillation of mineral oil C) Instillation of hot water B) Instillation of carbamide peroxide D) Use of a small forceps

Ans A

The nurse is assessing a client for objective symptoms of hearing difficulties. Which symptom leads the nurse to take alternate measures to ensure client understanding of teaching? A) The client interrupts by asking the nurse to repeat instruction. B) The client is quiet and responds appropriately. C) The client leans forward and turns the head. D) The client quietly reads the instructional literature.

Ans C Feedback: The nurse assesses objective symptoms of leaning forward and turning the head as symptoms of having difficulty hearing. The nurse would use alternate formats of teaching to reinforce key points. Asking to repeat information is a subjective indication of hearing difficulty. Responding appropriately and reading instructional literature does not indicate a hearing difficulty.

3. Which of the following actions should be taken when there is an insect in the ear? A) Instillation of mineral oil C) Instillation of hot water B) Instillation of carbamide peroxide D) Use of a small forceps

Ans: A

3. Which of the following nursing interventions should be included during the assessment of a client with an eye disorder? A) Check the extraocular muscles by instructing the client to keep his or her head still when following an object. B) Examine the retina with a direct ophthalmoscope. C) Use a tonometer to indent or flatten the surface of the eye. D) Instruct the client to stare at the central fixation spot on an Amsler grid and report if he or she sees any distortion of the squares.

Ans: A

4. Which of the following clients has an abnormal intraocular pressure (IOP)? A) A client with an IOP of 8 mm Hg C) A client with an IOP of 19 mm Hg B) A client with an IOP of 15 mm Hg D) A client with an IOP of 21 mm Hg

Ans: A

7. Which of the following clients have normal hearing acuity? A) A client who first perceives sound at 20 dB B) A client who first perceives sound at 40 dB C) A client for whom the painful sound occurs at 80 dB D) A client for whom the painful sound occurs at 100 dB

Ans: A

9. Which of the following is indicated by a diminished response in one eye during the caloric stimulation test? A) Inner ear disorder C) Outer ear disorder B) Middle ear disorder D) Age-related macular degeneration

Ans: A

26. Which of the following teaching items would be a priority in maintaining normal pressure range in the eye? A) Increase fiber in the diet. B) Avoid reading. C) East small meals. D) Treat allergy symptoms promptly.

Ans: A Feedback: Adding fiber to the diet will increase ease of bowel movements and prevent constipation and straining, which can inadvertently increase intraocular pressure. Eating small meals is insignificant in maintaining intraocular pressure. Avoid over-the-counter treatment of cold and allergy symptoms if contains cholinergic blockers. Reading is not significant in changing intraocular pressure, but eye strain should be avoided.

7. An eight-grade boy tells the school nurse that the eye doctor told him he had astigmatism and that meant his eyeball wasn't shaped right. The boy says he went home and looked in the mirror and both eyes looked just alike. What is the school nurse's best response? A) "Astigmatism means that the cornea of the eye is shaped differently than the cornea in most eyes." B) "Astigmatism means that the eye is shaped more like an olive than most eyes." C) "Astigmatism means that the inside of the eye is shaped differently than the inside of most eyes." D) "Astigmatism means that the lens of the eye is more of an oval shape than the lens in most eyes."

Ans: A Feedback: Astigmatism is visual distortion caused by an irregularly shaped cornea. Many people have both astigmatism and myopia or hyperopia. Options B, C, and D are incorrect because they are not the best answer.

1. A client is diagnosed with blepharitis. What symptoms should a nurse monitor in this client? A) Patchy flakes clinging to the eyelashes B) A red pustule in the internal tissue of the eyelid C) Redness surrounding the conjunctival sac D) A halo around the pupil

Ans: A Feedback: Blepharitis is an inflammation of the lid margins. The nurse monitors visible patchy flakes clinging to the eyelashes and about the lids. The condition does not cause redness or a halo around the pupil. In case of a sty, the nurse would observe a red pustule in the internal tissue of the eyelid.

11. The nurse is caring for a client experiencing hearing loss. The nurse uses the otoscope to assess the ear canal and tympanic membrane and notes a significant accumulation of cerumen. Which documentation of hearing loss type would be most accurate? A) Conductive B) Mixed C) Central D) Sensorineural

Ans: A Feedback: Conductive hearing loss occurs from an obstruction in the outer or middle ear such as from cerumen. Mixed hearing loss is a combination of conductive and sensorineural problems. Central hearing loss involves injury or damage to the nerves or the nuclei of the central nervous system. Sensorineural involves damage to the inner ear.

A nurse is assessing a pediatric client in a public health clinic. The parent states that the client has been sneezing and rubbing the eyes. The nurse looks at the client's eyes and documents objective symptoms of watery and red eyes. When reporting to the physician the assessment findings, which word is appropriate? A) Conjunctivitis B) Ptosis C) Nystagmus D) Proptosis

Ans: A Feedback: Conjunctivitis often stems from an allergy causing inflammation of the conjunctiva, which is a thin, transparent mucous membrane. Conjunctivitis can cause symptoms of itchiness, redness, and watery eyes. Ptosis is drooping of the upper eyelid. Proptosis is an extended and upper eyelid that delays in closing or remains partially open. Nystagmus is an uncontrolled oscillating movement of the eyeball.

A client is having problems with dizziness and complains of the "room spinning." The physician performs the caloric stimulation test. The nurse knows that a diminished response in one eye during the caloric stimulation testis indicative of what? A) Inner ear disorder B) Middle ear disorder C) Outer ear disorder D) Age-related macular degeneration

Ans: A Feedback: During the caloric stimulation test, a diminished response in one eye is significant for an inner ear disorder such as Ménière's disease. It does not signify a middle ear disorder, an outer ear disorder, or age-related macular degeneration.

15. While cleaning gutters, a client reports getting debris in the eyes. On inspection, no obvious foreign object is noted. Which of the following diagnostic evaluation techniques would be most beneficial for this client? A) Administer fluorescein dye. B) Obtain an x-ray for orbital fractures. C) Assess intraocular movements. D) Assess with tonometer.

Ans: A Feedback: Fluorescein dye stains the eye and helps to identify minute foreign body or abrasions in the cornea. X-ray of the eye orbit would be done if a blow to the area preceded the visit. Assessment of intraocular movements (cranial nerves III, IV, and VI) would not be indicated. Tonometry is done for assessment of intraocular pressure and would not be indicated.

18. A client has exhibited repeated return of hordeolum (sty). Which assessment finding is most important in determining care for this client? A) Use of mascara B) Low blood sugar C) Use of disposable wash cloths D) Antibacterial facial wash

Ans: A Feedback: Hordeolum is an infection usually caused by Staphylococcus aureus. To avoid transferring microorganisms, the use of eye cosmetics should be eliminated or at least products frequently replaced. Clients with high blood sugar are more likely to develop hordeolum. Use of disposable wash cloths, antibacterial cleansers, and good hygiene practices are preventable techniques.

5. The nurse is caring for a client who underwent surgery for a retinal detachment. The surgery included the injection of an air bubble to promote contact between the retina and choroids. What position should the nurse keep the client in? A) With the face parallel to the floor B) With the client's head slightly elevated C) With the client lying in a high Fowler's position D) With the client in an upright position

Ans: A Feedback: If an air bubble is instilled to promote contact between the retina and sclera, the client is positioned with the face parallel to the floor so that the bubble floats to the posterior of the eye. The client is asked to be on complete bed rest for several days with the head immobilized and to avoid any physical movements.

2. A client comes to the walk-in clinic complaining of a "bug in my ear." What action should be taken when there is an insect in the ear? A) Instillation of mineral oil B) Instillation of carbamide peroxide C) Instillation of hot water D) Use of a small forceps

Ans: A Feedback: Mineral oil is instilled into the ear to smother an insect. Carbamide peroxide is used to soften dried cerumen, and small forceps are used to remove solid objects. Hot liquids cause dizziness and should not be instilled in the ear.

20. The nurse is obtaining a history from a client complaining of ear pain and dizziness. Which assessment finding is the best evidence that the client has a perforated eardrum? A) Fluid draining in the external canal B) Pain has resolved C) Elevated white blood cell count D) Inflammation and a reddened eardrum

Ans: A Feedback: Noting the actual fluid in the ear canal alerts the nurse to the fact that there is a perforation in the tympanic membrane. The other options are also signs of a perforation but also signs of otitis media without perforation.

22. The nurse is instilling an antibiotic solution into the ear of an adult with otitis media. Which nursing action is most correct to ensure that the medication travels down the ear canal? A) Pull the auricle upward and back to instill the medication. B) Place a cotton ball in the ear canal to keep the medication in place. C) Pull the auricle downward and back to instill the medication. D) Use a cotton tip applicator to spread the medication deep in the canal.

Ans: A Feedback: The best nursing action is to straighten the ear canal by pulling the auricle upward and back. This action allows the medication to progress down the canal. In a child, the auricle should be pulled downward and back. The child's canal is straighter than the adults. The cotton ball will keep the medication in place but does not ensure that the medication in traveling down the canal. A cotton tip applicator should not be placed into the ear canal because it could perforate the eardrum.

The nurse is obtaining a visual history from a client who has noted an increased in glare and changes in color perception. Which assessment would the nurse anticipate to confirm a definitive diagnosis? A) Identification of opacities on the lens B) Identification of white circle around the cornea C) Identification of yellowish aging spot on the retina D) Identification of redness of the sclera

Ans: A Feedback: The client states an increased glare and changes in color perception, which indicates a cataract. Identification of opacities on the lens confirms that diagnosis. A white circle around the cornea and a yellowish aging spot are also symptoms of aging but with different symptoms. Redness of the sclera indicates irritation.

23. A 24-year-old female client is diagnosed with otosclerosis. Which teaching is most accurate? A) Symptoms may be accelerated by pregnancy. B) Medications can interfere with birth control pills. C) Menstrual periods may be longer and more severe. D) Females otosclerosis is linked with infertility.

Ans: A Feedback: The etiology of otosclerosis is unknown; however, it is more common in females than males and usually occurs in the second or third decade of life. It is accurate to instruct females that symptoms of otosclerosis seem to be accelerated during pregnancy.

The nursing student hopefuls are taking a pre-nursing anatomy and physiology class. What anatomical structure equalizes air pressure in the middle ear? A) Eustachian tube B) The malleus C) The pinna D) The meatus

Ans: A Feedback: The eustachian tube extends from the floor of the middle ear to the pharynx and is lined with mucous membrane. It equalizes air pressure in the middle ear. Options B, C, and D do not equalize pressure in the middle ear.

The nurse is caring for a client who just returned from a trip requiring an airline flight. The client commented on how his ears hurt upon descent. The nurse is correct in stating which site as being the pressure equalizer in the ear? A) Eustachian tube B) Auricle C) Tympanic membrane D) Labyrinth

Ans: A Feedback: The eustachian tube extends from the floor of the middle ear to the pharynx. It equalizes air pressure in the middle ear. The auricle is the fleshy portion of the outer ear which funnels sound waves to the inner ear. The tympanic membrane is the eardrum. The labyrinth is the inner ear which contains fluid.

The nurse is assessing client's eyes as part of the inspection part of the assessment process. For which reason does the nurse identify a normal variation in the angle of the lateral and medial canthus? A) Ethnic differences B) Chromosomal differences C) Structural changes D) Cosmetic alterations

Ans: A Feedback: The line between the lateral and medial canthus is usually horizontal. Variations are noted abnormally in children with Down syndrome and normally in individuals of Asian descent. Structural changes and cosmetic variations are not considered normal changes noted on assessment.

You are doing hearing tests at the local junior high school. Which of the following indicates normal hearing in a child? A) A client who first perceives sound at 20 dB B) A client who first perceives sound at 40 dB C) A client for whom the painful sound occurs at 80 dB D) A client for whom the painful sound occurs at 100 dB

Ans: A Feedback: The lowest level of sound that normal persons may first perceive is 20 dB. The painful sounds occur at 120 dB. The hearing acuity is determined by measuring the intensity at which a person first perceives sound.

32. The nurse is instructing a client's family members on the most incapacitating symptom of Ménière's disease. Which nursing instruction associated with the symptom is most helpful? A) Assist the client when ambulating. B) Keep a bucket beside the bed. C) Ensure low lighting in the room. D) Sit in front of the client when speaking.

Ans: A Feedback: The most incapacitating symptom of Ménière's disease is vertigo. When the client is experiencing vertigo or dizziness, the gate is unsteady. Having a person assist the client when ambulating is most helpful in preventing falls. Keeping a bucket at the bedside is helpful if the client is experiencing nausea. Photophobia is not a main symptom of Ménière's disease. If the client experiences hearing loss, being able to see the client's lips may be helpful.

36. The nurse is caring for a client diagnosed with an acoustic neuroma. Which assessment finding does the nurse anticipate when receiving shift report that is not related to hearing? A) Impaired facial movement and numbness and tingling B) Stroke like symptoms with bilateral facial droop C) Difficulty swallowing D) Inability to smell scents

Ans: A Feedback: The nurse anticipates that the previous shift nurse will report assessment findings of symptoms of impaired facial movement and possibly symptoms of facial numbness and tingling. Symptoms related to hearing include hearing loss, tinnitus, and vertigo.

The nurse is evaluating the client while taking the color vision test. Which response would the nurse anticipate when caring for a client with normal color vision? A) The nurse would anticipate the client identifying numbers and shapes. B) The nurse would anticipate a cross-eyed appearance. C) The nurse would anticipate responding to the color names in the pictures. D) The nurse would anticipate no differentiation in between colors.

Ans: A Feedback: The nurse is correct to anticipate the client being able to identify numbers and shapes dictated by different color codes. The other options do not test for color vision or indicate an inability to differentiate colors.

22. The client with chronic open-angle glaucoma is receiving timolol (Timoptic) eye drops. Which evaluation finding would indicate to the nurse the treatment is working? A) Intraocular pressure 15 mm Hg B) Reduced peripheral vision C) Halos around lights D) Decrease in nausea and vomiting

Ans: A Feedback: Timoptic is a beta-blocker that is used topically to decrease the flow rate of aqueous humor in the eye. As flow rate decreases, the intraocular pressure decreases. IOP of 12 to 21 mm Hg is within normal range. Reduced peripheral vision, halos around lights, and blurred vision are all symptoms of open-angle glaucoma. Nausea and vomiting are more likely to occur with acute angle-closure glaucoma.

27. An elderly client is scheduled for cataract surgery and asks the nurse, "Will I need to wear pop-bottle lenses after surgery?" Which is the most appropriate response from the nurse? A) "An implanted lens has replaced the need for corrective glasses." B) "Contact lenses are preferred by most clients after this surgery." C) "They can make corrective lenses much thinner now." D) "No lens is necessary with cataract surgery."

Ans: A Feedback: Vision is usually restored after cataract surgery with an intraocular lens implant. Contact lenses can be used but can be burdensome for the elderly. Corrective glass lenses can cause a distortion of peripheral vision and only required one lens (over operative eye). To restore vision after cataract surgery, a lens is required.

A client presents to the emergency room with symptoms of blurred vision. Which type of question would be best to ask first? A) "Have you ever had these symptoms before?" B) "Did these symptoms come on abruptly?" C) "Do you have a family history of vision problems?" D) "Do you have any other diseases?"

Ans: A Feedback: When a client presents with unusual symptoms, a first question assesses if he or she has ever had these symptoms before. This prepares a starting place for the assessment. If the client did have these symptoms before, questions regarding the specific nature and similarities of that experience guide the assessment.

9. A nursing instructor is teaching pre-nursing students in a pathophysiology class. What would the instructor teach the students about Ménière's disease? A) It is referred to as endolymphatic hydrops. B) It originates in the middle ear. C) It is referred to as lymphatic hydrops. D) It originates in the outer ear.

Ans: A Feedback: When a person moves his or her head, the endolymph also moves, and nerve receptors within the membranous labyrinth send signals to the brain about the movement. In Ménière's disease, an increase in endolymph causes the membranous labyrinth to dilate like a balloon; this is referred to as endolymphatic hydrops. Ménière's disease does not originate in either the middle or the outer ear, and it is not referred to as lymphatic hydrops.

25. An elderly client is admitted with the diagnosis of retinal detachment and is scheduled for laser surgery and scleral buckling procedure. The nurse anticipates which of the following symptoms to be exhibited in this client? Select all that apply. A) Flashing lights B) Cobwebs in vision field C) Complete loss of vision in both eyes D) Loss of central vision E) Eye pain F) Arcus senilis

Ans: A, B Feedback: Many clients with detached retina experience a sensation of a curtain or veil lowering over vision field, flashing of lights, floaters, cobwebs, or spots. Complete vision loss can occur in the affected eye. Loss of central vision, eye pain, and arcus senilis is not indicated in this disorder.

24. The nurse is employed in an ear, nose, and throat (ENT) physician's office, obtaining a client history. The nurse documents the following client statements. Which symptoms may indicate a diagnosis of otosclerosis? Select all that apply. A) "It seems that I increasingly could not hear my kids talk to me." B) "I woke up on Monday and had ear pain with a marked decrease in hearing." C) "I now notice a ringing in my ears especially when I lay down to sleep at night." D) "I can hear better when someone speaks in low tones." E) "I can hear best when you put the tuning fork behind my ear."

Ans: A, C, E Feedback: Although the cause of otosclerosis is unknown, there are specific symptoms because of the interference of the vibrations in the ear. The symptoms of diagnosis of otosclerosis include a progressive, bilateral hearing loss; tinnitus especially noted at night; and the outcome of the Rinne test as being that sound is heard best with the tuning fork behind the ear. The nurse would not identify an abrupt onset of hearing loss or improvement of hearing in low tones.

15. The nurse is obtaining subjective data from a client with difficulty hearing. In order to assist the client in hearing the nurse's voice, which adjustments are made? Select all that apply. A) Speak in a clear voice B) Use high-pitched tones C) Clearly articulate D) Speak in a louder volume E) Speak in a lower tone F) Face the client when speaking

Ans: A, C, E, F Feedback: To obtain data from the client, the nurse must be able to communicate with the client. In a client with hearing difficulty, the nurse is correct to speak in a clear voice and articulate the words clearly. Also, speaking in lower tones is better for hearing. Facing the client enables the client to focus on interpreting lipreading to compare with hearing the words. High-pitched tones and speaking in a loud volume are more difficult for word interpretation.

When caring for a client with a foreign object removed from the eye, the nurse is most correct to assess the eye protective functions of which structures? Select all that apply. A) Eyelids and lashes B) Aqueous humor C) Superior and inferior oblique muscles D) Conjunctiva E) Sclera F) Tears

Ans: A, F Feedback: The nurse is correct to assess the eyelids and lashes and also tears as the protective structures. The eyelids protect against foreign bodies and adjust the amount of light that enters the eye. The eyelashes trap foreign debris. Periodic blinking clears dust and particles from the surface of the eyes. The aqueous humor and sclera are intraocular structures. Oblique muscles move the eye left and right. The conjunctiva is a sensitive transparent mucous membrane that alerts the individual to a foreign object in the eye.

1. Which of the following advice should the nurse give a self-conscious client with hearing loss to protect his or her self-esteem? A) Pretend to follow conversations by nodding the head. B) Be forthright and inform others about the hearing deficit. C) Follow lip movements closely. D) Avoid excess socializing.

Ans: B

10. An older client is observed holding reading materials at an increasing distance to focus properly. Which of the following age-related changes does this indicate? A) Cataract B) Presbyopia C) Myopia D) Macular degeneration

Ans: B

2. Which of the following assessments should the nurse observe in a client with a cataract? A) A burning sensation and the sensation of an object in the eye B) Blurred or cloudy visual image C) Inability to produce sufficient tears D) A swollen lacrimal caruncle

Ans: B

5. During a Weber test, where should the tuning fork be placed? A) On the mastoid process behind the ear B) In the midline of the client's skull or in the center of the forehead C) Near the external meatus of each ear D) Under the bridge of the nose

Ans: B

5. Which of the following is the most characteristic symptom of otosclerosis? A) The client being distressed in the mornings B) A progressive, bilateral loss of hearing C) A red and swollen ear drum D) The client describing a history of having had a recent upper respiratory infection

Ans: B

6. Which of the following tests should be recommended to a client with a loss of balance? A) Audiometric test B) Romberg test C) Weber test D) Rinne test

Ans: B

9. Which of the following symptoms might a client with Ménière's disease exhibit? A) Pinkish-orange eardrum B) Nystagmus of the eyes C) Enlarged lymph nodes behind the ear D) Swelling and redness in the auditory canal

Ans: B

8. You are teaching a parent how to instill drops in their 12-year-old son's eyes. Which action would you teach the parent is accomplished first? A) Close the eye gently. B) Tilt the head slightly backward. C) Instill the prescribed number of drops into the conjunctival pocket. D) Do not allow the tip of the container to touch the eye.

Ans: B Feedback: To instill eye drops, tilt the head slightly backward and toward the eye in which the medication is to be instilled. Do not allow the tip of the container to touch the eye. Instill the prescribed number of drops into the conjunctival pocket or apply a thin ribbon of ointment directly into the conjunctival pocket, beginning at the inner corner and moving outward. Close the eye gently. Options A, C, and D are not the first action in instilling eye drops.

3. You are teaching a class on diseases of the ear. What would you teach the class is the most characteristic symptom of otosclerosis? A) The client being distressed in the mornings B) A progressive, bilateral loss of hearing C) A red and swollen ear drum D) The client describing a recent upper respiratory infection

Ans: B Feedback: A progressive, bilateral loss of hearing is the characteristic symptom of otosclerosis. Tinnitus appears as the loss of hearing progresses; it is especially noticeable at night, when surroundings are quiet, and may be quite distressing to the client. The eardrum appears pinkish-orange from structural changes in the middle ear. The client often describes a history of having had a recent upper respiratory infection in case of otitis media, not otosclerosis.

37. The nurse is caring for a client postoperatively after removal of an acoustic neuroma using surgery. Which client symptom does the nurse related to the physician? A) Temperature of 100.2° F and discomfort B) Restlessness and confusion C) Redness and inflammation at incision site D) Hearing loss and discomfort

Ans: B Feedback: After surgery, the nurse closely monitors the client for increased intracranial pressure including restlessness and confusion. The physician would be notified immediately if symptoms occurred. The physician would not be notified of the other options because they are either expected following surgery or not significant at the present time.

13. The occupational nurse is advising a customer service representative client on assistive devices for hearing because the client has progressive hearing loss. In discussing the options with the client, which type would be the last option offered by the nurse? A) Battery-operated hearing aid B) American sign language C) Headsets with amplifiers D) Text-based telecommunications

Ans: B Feedback: Although the American sign language is an asset to use for communication, a client with an occupation of customer service representative needs accommodations to be able to understand the spoken word.

17. A client is diagnosed with uveitis. Which assessment finding is most important in determining likelihood of recurrence? A) Chemical exposure B) Ankylosing spondylitis C) Glaucoma D) Extended contact use

Ans: B Feedback: Although the cause of uveitis is unknown, it is detected with frequency among clients with autoimmune disorders (such as ankylosing spondylitis). Chemical exposure and extended contact use are not indicated with uveitis. Glaucoma can be a complication of uveitis.

3. A client is diagnosed with keratitis. What advice should the nurse give this client? A) Use warm soaks frequently. B) Use dark glasses. C) Wash the face and hair frequently. D) Massage the surrounding area.

Ans: B Feedback: Dark glasses are recommended for a client with keratitis to relieve photophobia. Treatment for keratitis does not require use of warm soaks and massages or washing of the face and hair.

7. There are several types of hearing loss. Which type of hearing loss benefits most from the use of a hearing aid? A) Sensorineural B) Conductive C) Genetic D) Acquired

Ans: B Feedback: Diminished hearing results from a conductive loss, sensorineural loss, or both. Clients with a conductive hearing loss benefit more from the use of a hearing aid because the structures that convert sound into energy and facilitate perception of sound in the brain continue to function. Genetic and acquired are not types of hearing loss.

9. A client has just been diagnosed with glaucoma. What teaching should the nurse include with this client? A) How long they have to wear dark glasses B) Maintain regular bowel habits C) What vegetables to eat D) When they can read again

Ans: B Feedback: Instructions for the client with glaucoma include the following: Obtain assistance from a family member, relative, or friend if you have trouble instilling eye drops. Avoid all drugs that contain atropine. Check with physician or pharmacist before using any nonprescription drug. preparations for cold or allergy symptoms may contain an atropine-like drug. Maintain regular bowel habits; straining at stool can raise IOP. Avoid heavy lifting and emotional upsets (especially crying) because they increase IOP. Eating vegetables and reading do not increase IOP.

The nurse is assessing a client's hearing using the Rinne test. When providing instruction to elicit client feedback, which instruction is essential? A) Raise your hand when you hear the vibration. B) Raise your hand when you no longer hear sound. C) Raise your hand when the vibration exceeds the sound. D) Raise your hand when the sound exceeds the vibration.

Ans: B Feedback: It is essential to provide clear directions on when the client is to notify the nurse of client response. The information gleaned from the response is what the nurse uses to interpret the test. The correct time to induce feedback is when the vibration is held.

5. A nursing student is presenting a report on Ménière's disease to other members of the class. What symptom would the student list? A) Pinkish-orange eardrum B) Nystagmus of the eyes C) Enlarged lymph nodes behind the ear D) Swelling and redness in the auditory canal

Ans: B Feedback: Nystagmus of the eyes may occur in a client with Ménière's disease caused by an imbalance in vestibular control of eye movements. Pinkish-orange eardrums, enlarged lymph nodes, or swelling and redness in the auditory canal are not observed in a client with Ménière's disease.

35. The nurse is caring for a client with symptoms of ototoxicity from aminoglycoside administration. On which structure does the medication produce the ototoxic effect? A) The auditory canal B) The eighth cranial nerve C) The tympanic membrane D) The cochlear nerve

Ans: B Feedback: Ototoxicity describes the detrimental effect of aminoglycosides on the eighth cranial nerve. Signs and symptoms include tinnitus and sensorineural hearing. The other options are not related to the ototoxic effects.

11. Following an ophthalmologic exam, an anxious client asks the nurse, "How serious is a refraction error?" Which of the following is the best response from the nurse? A) "It is nothing serious." B) "It means corrective lenses are required." C) "Simple surgery can fix this problem." D) "This is normal for anyone your age."

Ans: B Feedback: Refractive errors can be corrected with glasses or contact lenses. Telling a client that "nothing is serious" does not provide the necessary information to help alleviate fears. The word surgery can increase fears. If the refractive error is associated with aging, this is a normal finding but does not provide information to the condition.

28. Which nursing goal is a priority when caring for a client newly diagnosed with vertigo? A) Patient will maintain therapeutic medication schedule. B) Patient will remain safe while ambulating in the home. C) Patient will have a caretaker with him or her in the home. D) Patient will closes eyes as needed to reduce symptoms.

Ans: B Feedback: Safety is always a concern when a client is experiencing vertigo. The goal of the nurse's instruction and care is for the client to remain safe. Maintaining a therapeutic medication schedule and caretaker and establishing strategies to reduce symptoms are important but not of highest priority.

12. The nurse is obtaining a history from a client who indicates hearing loss due to drug toxicity. Which type of hearing loss is noted? A) Conductive B) Sensorineural C) Mixed D) Central

Ans: B Feedback: Sensorineural hearing loss involves damage to the inner ear from etiologies including drug toxicity. Conductive hearing loss occurs from an obstruction in the middle to outer ear. Mixed hearing loss includes a combination of conductive and sensorineural hearing loss. Central hearing loss involves injury or damage to the nerves or the nuclei of the central nervous system.

The client is having a Weber test. During a Weber test, where should the tuning fork be placed? A) On the mastoid process behind the ear B) In the midline of the client's skull or in the center of the forehead C) Near the external meatus of each ear D) Under the bridge of the nose

Ans: B Feedback: The Weber test is performed by striking the tuning fork and placing its stem in the midline of the client's skull or in the center of the forehead. In the Rinne test, the tuning fork is struck and placed on the mastoid process behind the ear. The tuning fork is not placed near the external meatus of each ear or under the bridge of the nose.

21. A client splashes bleach into the right eye and requires irrigation of the eye. Which nursing action is most important to prevent extension of chemical irritation? A) Use only saline solution. B) Turn head with right side lower than the left. C) Tilt head backward and irrigate both eyes. D) Direct solution toward the nasolacrimal duct.

Ans: B Feedback: The chemical in the right eye can drain into the left eye causing additional damage. To avoid the drainage from affected eye to unaffected eye requires careful positioning during irrigation. Saline solution is usually used, but water can be used for emergency flushing.

26. The nurse is evaluating the independent care of a client recovering from a stapedectomy. Which action, made by the client, indicates a need for further teaching? A) The client turns head slowly when family approaches. B) The client uses clean technique to clean the wound. C) Taking antibiotics on a convenient schedule D) Ensure assistance upon ambulation.

Ans: B Feedback: The client needs further instructions on using aseptic technique when completing wound care. Using aseptic technique reduces the introduction and transmission of microorganisms and protects the client from introduction of pathogens. The other options are correct.

The client is consulting with a physician regarding a potential diagnosis of Ménière's disease. The nurse is assisting in positional testing and documentation. Which diagnostic test would the nurse anticipate to obtain a more precise evaluation of vestibular function? A) Audiometry B) Electronystagmography C) Caloric stimulation test D) Romberg test

Ans: B Feedback: The electronystagmography is a more precise method for evaluation vestibular function. It is performed in conjunction with caloric stimulation. When the fluid is instilled in the ear, a machine records the duration and velocity of eye movements with electrodes attached around the eye. Audiometry measures hearing acuity. The Romberg test measures balance.

21. The nurse is caring for a client with recurrent ear infections. The nurse assesses the client for further infectious processes traveling deeper into the tissue and becoming more lethal. Which infection, originated in the ear, is of most concern? A) Mastoiditis B) Meningitis C) Sinusitis D) Labyrinthitis

Ans: B Feedback: The infection stemming for the ear may extend to the meninges, causing meningitis, or a brain abscess could occur. This could be life threatening. The other options are also potential complications of an ear infection.

25. Which nursing suggestion would be most helpful to the client with recurrent otitis externa? A) Use a cotton applicator to ensure that the ear canal is dry. B) Place ear plugs into the ears before swimming. C) Flush the ear with hydrogen peroxide. D) Avoid lying on the side of the affected ear.

Ans: B Feedback: The nurse instructs the client to carry out the medical treatment and provides health teaching to prevent recurrence. For example, he or she advises swimmers to wear soft plastic ear plugs to prevent trapping water in the ear. A cotton tip applicator should not be placed into the ear canal because it could perforate the eardrum. Above all, the nurse advises the client to avoid the use of nonprescription remedies unless they have been approved by the physician and to contact the physician if symptoms are not relieved in a few days.

The nurse is establishing a visual test using the Snellen chart for a client experiencing visual changes. At which distance should the nurse instruct the client to stand? A) A 10-feet distance B) A 20-feet distance C) A 30-feet distance D) A 40-feet distance

Ans: B Feedback: The nurse is correct in instructing the client to stand at a 20-feet distance from the Snellen chart. Often, the nurse places tape on the floor to denote the correct distance for the client to stand.

The nurse is assisting the eye surgeon in completing an examination of the eye. Which piece of equipment would the nurse provide to the physician to examine the fundus and interior of the eye? A) Retinoscope B) Ophthalmoscope C) Tonometer D) Amsler grid

Ans: B Feedback: The nurse is correct to provide an ophthalmoscope to the surgeon for examination of the fundus or interior of the eye. A retinoscope is used to determine errors in refraction. A tonometer measures intraocular pressure. An Amsler grid tests for problems with the macula.

20. When caring for a client with progressive macular degeneration, which teaching measure is primary for client safety? A) Patch the affected eye. B) Turn head side to side when walking. C) Avoid bending over. D) Avoid straining the eyes.

Ans: B Feedback: To expand the visual field, the client should be taught to turn the head from side to side when walking. This effort can assist in improving vision field and decrease risk of injury from tripping and falls. This technique helps to maximize the partial sight. A patch may assist with symptoms of dizziness associated with hemianopia but not significant for safety. Bending over can increase intracranial pressure but not significant with macular degeneration. Straining the eyes with close work and reading can increase blurring of vision in macular degeneration but not significant for safety.

33. The nurse is assisting the client in planning care during exacerbations of Ménière's disease. Which diet would the nurse identify as appropriate at this time? A) A high-protein diet B) A low-sodium diet C) A low-fat diet D) A calorie-controlled diet

Ans: B Feedback: Treatment for Ménière's disease is related to reducing fluid production in the inner ear, facilitating its drainage, and treating the symptoms that accompany the attack. A low-sodium and sodium-free diet lessens edema.

A client, diagnosed with a cataract, comes into the clinic. What assessment should the nurse observe in this client? A) A burning sensation and the sensation of an object in the eye B) Blurred or cloudy visual image C) Inability to produce sufficient tears D) A swollen lacrimal caruncle

Ans: B Feedback: When a cataract forms, the light is blocked from reaching the macula, and the visual image becomes blurred or cloudy. The client does not experience any burning or the sensation of an object in the eye, an inability to produce sufficient tears, or a swollen lacrimal caruncle.

29. The nurse on a cruise ship is assessing clients for motion sickness. Which of the following is a common misconception? A) Repeated motion is the cause. B) Once symptoms occur, they will always be present. C) Medications help the symptoms. D) Pallor and diaphoresis is a first symptom.

Ans: B Feedback: When the client experiences motion sickness, the client will use that data to avoid further symptoms in the future. The client can use medication, change location or position, and recognize symptoms earlier for symptom management. The other options are correct and teachable statements.

17. The nurse is supervising a family member who instilling ear drops into the client's ear. Which of the following statements, made by the family member, would require further nursing instruction? A) "Turn your head to the side so I can put these drops in." B) "These drops are cold from being on the window seal." C) "Let me put this cotton ball in your ear because I put the drop in." D) "I squeeze the dropper to put a drop of medicine in the ear."

Ans: B Feedback: When the family member states that the drops are cold, the nurse would encourage the family member to place the bottle in a warm bath or warm the bottle in their hands. Cold or hot liquids, instilled in the ear, may cause dizziness and potential for injury.

10. Which of the following aspects should the nurse closely monitor for in clients who have been administered salicylates, loop diuretics, quinidine, quinine, or aminoglycosides? A) Signs of hypotension C) Tinnitus and sensorineural hearing loss B) Reduced urinary output D) Impaired facial movement

Ans: C

4. Which of the following nursing actions is important for a client who has undergone myringotomy? A) Ensure that the client has plenty of fluids. B) Ask the client to avoid coughing or sneezing. C) Wipe the external ear with a dry, sterile cotton applicator. D) Advise the client to wear a hat with earflaps, or a scarf, at all times.

Ans: C

8. Why is it important for a nurse to advise a client who has undergone a stapedectomy to refrain from blowing the nose? A) It may cause sudden headaches. C) It may dislodge the prosthesis. B) It may cause vertigo. D) It may cause excessive drainage.

Ans: C

Why should the nurse advise periodic blinking to a client who works in a dusty outdoor environment? A) To control the amount of sunlight that enters the eye B) To minimize the impact of the wind on the eye and to trap foreign debris C) To clear the dust and particles from the surface of the eyes D) To prevent the collection of tears over the surface of the eye

Ans: C

14. Which technique would be most beneficial for ambulation of a client who is visually impaired? A) Speak before touching the client. B) Provide a detailed description of the room and walkway. C) Allow client to follow your lead. D) Provide the client with a see-eye guide dog.

Ans: C Feedback: A blind person feels more secure and safe when assisted by someone who is sighted. The nurse should walk slightly ahead while allowing the client to hold onto the upper arm or elbow of the nurse. Speaking before touching is an important care item in dealing with clients who have impaired vision but does not assist in ambulation. Providing a detailed description of the room may allow the client an image of the surroundings but not as helpful in initial ambulation. Finding a perfect fit between guide dog and client is a lengthy process and should be pursued upon request of client.

The nurse is assisting with the administration of a caloric stimulation test. Which client response would the nurse document as an expected response? A) Dizziness B) Headache C) Nystagmus D) Double vision

Ans: C Feedback: A caloric stimulation test assesses vestibular reflexes of the inner ear that control balance. Warm or cool water or air is instilled into the external meatus of the ear separately. Nystagmus, a quivering movement of the eyes, is the expected response. Slight dizziness may be experienced but is not the expected response. Headache and double vision is not the expected response.

30. Miotic eye solutions are often ordered in the treatment of glaucoma. Which is the best nursing rationale for the use of this medication? A) Constricts intraocular vessels B) Paralyzes ciliary muscles C) Constricts pupil D) Dilates the pupil

Ans: C Feedback: A miotic agent works by constricting the pupil and pulling the iris away from the drainage channels so that the aqueous fluid can escape. These medications increase outflow and decrease intraocular pressure. Cycloplegics paralyze the ciliary muscles of the eye. Mydriatics drugs are used to dilate the pupil and are contraindicated in glaucoma.

The nurse is instructing a nursing student when a new client comes to the eye clinic. The client explains that he thinks he has a corneal abrasion. The nurse should explain what to the student nurse? A) "To detect corneal abrasions, an ophthalmoscope is used." B) "To detect corneal abrasions, ultrasonography is used." C) "To detect corneal abrasions, a slit lamp is used." D) "To detect corneal abrasions, retinal angiography is used."

Ans: C Feedback: A slit lamp is a binocular microscope that magnifies the surface of the eye. A beam of light, narrowed to a slit, is directed at the cornea, facilitating an examination of structures and fluid in the anterior segment of the eye. This examination is used to identify disorders such as corneal abrasions, iritis, conjunctivitis, and cataracts. Options A, B, and D are not used to detect corneal abrasions.

4. A nurse is doing preoperative and postoperative teaching with a client who is undergoing cataract surgery. What is an important teaching point the nurse should teach the client about? A) Feelings of depression B) Increased urine output C) Eat soft, easily chewed food until healing is complete D) Development of a "black" eye

Ans: C Feedback: Advise clients who have had cataract surgery to eat soft, easily chewed foods until healing is complete to avoid tearing from excessive facial movements. Clients who undergo cataract surgery do not become depressed, have increased urine output, or develop a "black" eye.

Audiometry is testing that measures hearing acuity precisely. Who does the nurse know can perform audiometric testing? A) School nurse B) Hearing aide salesperson C) Audiologist D) Office nurse

Ans: C Feedback: Audiometry is done by an audiologist. Audiometric testing measures hearing acuity precisely. Options A, B, and D can screen hearing, but they cannot do audiometric testing.

A client has been referred to an ophthalmologist for suspected macular degeneration. The nurse knows to prepare what test for the physician to give the client? A) Ishihara polychromatic plates B) Visual field C) Amsler grid D) Slit lamp

Ans: C Feedback: Clients with macular problems are tested with an Amsler grid. It is made up of a geometric grid of identical squares with a central fixation point. The examiner instructs the client to stare at the central fixation spot on the grid and report if they see any distortion of the squares. Clients with macular problems may say some of the squares are faded or wavy. An Ishihara polychromatic plate, visual field, or slit lamp test will not diagnose macular degeneration.

28. Immediately following cataract removal, which symptom would be most alarming to the nurse? A) Irritation in the operative eye B) Dilation of the pupil C) Dry, tickling cough D) Fever

Ans: C Feedback: Coughing can rise the intraocular pressure and should be avoided. A cough suppressant can be prescribed. The pupil was intentionally dilated during the surgical approach and will resolve. The client may complain of mild eye irritation in the immediate postoperative period. Fever can be a complication of cataract surgery but not expected in the immediate period postoperatively.

34. The nurse is caring for a client being treated for Ménière's disease. Which medication is monitored closely due to its addictive properties? A) Meclizine (Antivert) B) Hydrochlorothiazide C) Diazepam (Valium) D) Promethazine (Phenergan)

Ans: C Feedback: Diazepam (Valium) is used to treat the client with Ménière's disease to help control vertigo. Diazepam is a tranquilizer that has addictive properties. The other options, used in the treatment of Ménière's disease, do not have any or significant addictive properties.

6. During a pharmacology class, the students are told that some drugs need to be closely monitored. What aspect should the nurse closely monitor for in clients who have been administered salicylates, loop diuretics, quinidine, quinine, or aminoglycosides? A) Signs of hypotension B) Reduced urinary output C) Tinnitus and sensorineural hearing loss D) Impaired facial movement

Ans: C Feedback: It is important that nurses are knowledgeable about the ototoxic effects of certain medications such as salicylates, loop diuretics, quinidine, quinine, and aminoglycosides. Signs and symptoms of ototoxicity include tinnitus and sensorineural hearing loss. Hypotension, reduced urinary output, and impaired facial movement are not signs of ototoxicity.

23. A client with chronic open-angle glaucoma is now presenting with eye pain and intraocular pressure of 50 mm Hg. An immediate iridotomy is scheduled. Which of the following describes the desired effects of this procedure? A) Reverse optic nerve damage B) Restore vision C) Improve outflow drainage D) To relieve pain

Ans: C Feedback: Laser iridotomy or standard iridotomy is a surgical procedure that provides additional outlet drainage of aqueous humor. This is done to lower the IOP as quickly as possible since permanent vision loss can occur in 1 to 2 days. Once optic nerve damage occurs, it cannot be reversed, and vision is not restored. Pain that occurs with rising IOP will be controlled once pressure is lowered through improved outflow drainage.

1. A client comes to the occupational health nurse complaining of eye irritation. The client works in a dusty, outdoor environment. Why should the nurse advise periodic blinking to this client? A) To control the amount of sunlight that enters the eye B) To minimize the impact of the wind on the eye and to trap foreign debris C) To clear the dust and particles from the surface of the eyes D) To prevent the collection of tears over the surface of the eye

Ans: C Feedback: Periodic blinking clears the dust and particles from the surface of the eyes. The eyelids also spread tears over the surface of the eye, which helps bathe and lubricate the surface. The eyelids protect against foreign bodies and adjust the amount of light that enters the eye, whereas the eyelashes trap foreign debris.

12. A middle-aged client reports increasing difficulty reading the newspaper print. Which of the following nursing explanations best describes this type of refractive error? A) Client is nearsighted. B) Lens has become cloudy and thick. C) Loss of elasticity of the ciliary processes D) Floaters in the eye increase with age.

Ans: C Feedback: Presbyopia is a result of poor accommodation due to a loss of elasticity of the ciliary muscles and lens. Nearsighted refers to myopia. Cloudiness of lens is also associated with the aging process and does interfere with vision as a result of cataract formation. Floaters in the eyes are more apparent with aging but appear as dark spots.

8. What kind of otitis media is a pathogen-free fluid behind the tympanic membrane, resulting from irritation associated with respiratory allergies and enlarged adenoids? A) Purulent otitis media B) Infectious otitis media C) Serous otitis media D) Sterile otitis media

Ans: C Feedback: Serous otitis media, a collection of pathogen-free fluid behind the tympanic membrane, results from irritation associated with respiratory allergies and enlarged adenoids. Options B and D are distractors for this question. Purulent otitis media usually results from the spread of microorganisms from the eustachian tube to the middle ear during upper respiratory infections.

The nurse is caring for a client with increased fluid accumulation in the eye. When assessing the client, which structure within the eye is noted to drain fluid from the anterior chamber? A) Fovea centralis B) Canthus C) Canal of Schlemm D) Choroid

Ans: C Feedback: The canal of Schlemm drains the anterior chamber of the eye. By draining the fluid, it decreases the fluid amount and pressure in the eye. The other options have no draining ability.

The nurse is obtaining a history on a client stating the inability to read the newspaper and even seeing detail when looking at an image. Which assessment test would add additional data for a diagnosis? A) Assess if the pupils are equal and reactive to light. B) Assess vision on the Snellen chart. C) Assess peripheral vision. D) Assess color vision.

Ans: C Feedback: The client states symptoms of the inability to discriminate letters, words, and details of an image, indicating the degeneration of the macula. If the macula is damaged, the client will only have the ability to see movement and gross objects in the peripheral fields. Assessing the peripheral vision will add essential information. The other visual tests are not as important at this time.

18. The nurse is caring for an 8-year-old and anticipates that the client has otitis externa from symptoms stated on the history. Which symptoms, from the history and physical examination, would confirm the diagnosis? A) Discomfort in the ear B) Difficulty hearing C) Pus noted in the ear canal D) Inflammation around the tympanic membrane

Ans: C Feedback: The diagnosis of otitis externa (inflammation of the tissue of the outer ear) is confirmed by the presence of pus in the ear canal. The inflammation is usually caused by an overgrowth of pathogens. The other symptoms are also common in otitis media.

16. The nurse is instructing the client with dried cerumen blocking the ear canal on potential methods to reduce symptoms. Which at home methods of cerumen removal is discouraged? A) Instilling 1 to 2 drops of half-strength peroxide in the ear B) Using warm glycerin or mineral oil to soften the cerumen C) Removing the cerumen by means of a cotton tip applicator D) Irrigating the ear with warm water and a rubber-bulb syringe

Ans: C Feedback: The nurse is an important resource person to consult when a client has an issue with the ear structure or hearing. The nurse is correct to discourage placing anything down the ear canal that could push the cerumen deeper toward or puncture the tympanic membrane. The other options are appropriate to soften and lubricate the cerumen or to irrigate the cerumen from the ear.

The nurse is caring for a client ordered multiple eye screening. Following which procedure will the nurse instruct the client on a yellow coloring to the skin and urine as being normal? A) Ultrasonography B) Retinal Imaging C) Retinal Angiography D) Retinoscopy

Ans: C Feedback: The nurse is most correct to instruct the client that his skin and urine may turn yellow following a retinal angiography. Sodium fluorescein is a water-soluble dye that is injected into a vein. The dye then travels to the retinal arteries and capillaries, where pictures are obtained of the vascular supply. The other options do not include a dye injection.

6. A client has undergone enucleation. What complication of enucleation should be addressed by the nurse? A) Hypotension B) Nausea and vomiting C) Hemorrhage D) Pneumonia

Ans: C Feedback: The nurse should take measures to prevent hemorrhage, a complication of enucleation, by applying a pressure dressing. Nausea and vomiting may be common side effects of surgery. Enucleation does not increase risk of developing hypotension or pneumonia.

The nurse is assisting in providing coordination of services between the physician's office and vision specialist's office for a client who is being referred for potential retinal surgery. Which eye care specialist will the nurse make the referral to? A) Optician B) Optometrist C) Ophthalmologist D) Ophthalmic technician

Ans: C Feedback: The ophthalmologist is a physician who performs surgery on clients with eye disorders. The optician makes eyeglasses or contact lenses. The optometrist tests vision and prescribes corrective lenses. The ophthalmic technician assists in selective eye tests and procedures.

31. Which of the following assessment findings is least helpful in identifying a cause of Ménière's disease? A) Family history B) Allergic reactions C) Food intolerance D) Head injury

Ans: C Feedback: There is no evidence that a food intolerance is an identifying cause of Meniere's disease. The other options are attributed to the disease.

10. You are caring for a client with open-angle glaucoma. You know that this disease causes which of the following? Select all that apply. A) Atrophy of nerve fibers in the central area of the retina B) Edema of the lens C) Degeneration of the optic nerve D) Edema of the cornea E) Atrophy of nerve fibers in the peripheral areas of the retina

Ans: C, D, E Feedback: Open-angle glaucoma occurs when structures in the drainage system (i.e., trabecular meshwork and canal of Schlemm) degenerate, and the exit channels for aqueous fluid become blocked. As the IOP rises, it causes edema of the cornea, atrophy of nerve fibers in the peripheral areas of the retina, and degeneration of the optic nerve. This makes options A and B incorrect.

2. Which of the following nursing actions is important when instilling ear drops to remove excessive cerumen in a client with impacted cerumen? A) Insert the irrigating syringe deeply. B) Direct the flow of the ear drops toward the eardrum. C) Refrigerate before instillation. D) Place the container in warm water before instillation.

Ans: D

6. Why does the nurse advise a low-sodium diet and smoking cessation to a client with Ménière's disease? A) To minimize the adverse effects of drug therapy B) To reduce the magnitude of the hearing deficit C) To minimize the risk of a tumor that involves the vestibulocochlear nerve D) To reduce the production of fluid in the inner ear

Ans: D

7. Which of the following is included during the assessment of a client with an acoustic neuroma? A) Measure the client's urine output. B) Note the client's height and weight. C) Test the client's ability to sustain balance. D) Test for facial sensation.

Ans: D

8. Which of the following should the nurse recommend to a client to soften hardened cerumen? A) Avoid harsh sunlight. C) Increase intake of beta-carotene. B) Increase intake of red meat. D) Take nonprescription preparations.

Ans: D

. The nurse is completing a corneal light reflex test using a penlight. Which result would indicate a normal test result? A) The pupils have reaction to light. B) The eyes follow the light in all four directions. C) The client can see the light using peripheral vision. D) The light reflection is in the same spot on each eye.

Ans: D Feedback: A normal corneal light reflex test is when the light reflex is even, reflecting the light at the same spot on both eyes. If the light reflex is uneven, it indicates deviated alignment of the eyes, possibly due to muscle weakness or paralysis.

10. You are caring for a client who is poststapedectomy. What would you include in your nursing care? A) Place the client on the operative side. B) Keep the affected ear packed with cotton. C) Encourage the client to exercise within 24 hours. D) Assess the facial nerve.

Ans: D Feedback: After surgery, the nurse positions the client on the nonoperative side. He or she takes care to prevent dislodgment of the prosthesis as a result of coughing, sneezing, or vomiting. Nausea and dizziness are common problems. The nurse assesses facial nerve function by checking symmetry when the client smiles or frowns. The nurse does not keep the ear packed with cotton or encourage the client to exercise.

19. The nurse is instructing the mother of an infant diagnosed with otitis media. The mother states, "Why is my child getting recurrent ear infections?" Which assessment question is best? A) "Do you cover the child's ears when going outdoors?" B) "Do you administer the child's vitamins on a daily basis?" C) "Do have other children with similar symptoms?" D) "Do you allow the infant to hold or prop the bottle during feeding?"

Ans: D Feedback: Allowing the bottle to be held by the infant or propped by a towel, enables the formula to leak/drip from the child's mouth and flow to the ears where the formula can proceed down the eustachian tube, causing otitis media. It is a common practice to cover the child's ears and take daily vitamins. An ear infection is not typically a contagious disease process.

24. Which assessment finding would contraindicate the use of atropine in a client scheduled for general anesthesia? A) Detached retina B) Cerebrovascular accident C) Cataracts D) Glaucoma

Ans: D Feedback: Cholinergic blockers (such as atropine) are often used preoperatively to dry up secretions. Use of these drugs results in dilation of pupils, which increases IOP. Clients with glaucoma should avoid use of atropine in an effort to maintain normal range of IOP. Detached retina, CVA, and cataracts are insignificant in the use of cholinergic blockers.

16. The nurse is instructing the client on use of ophthalmic eye ointment for treatment of an eye disorder. The ointment is ordered once daily. When is the best time to apply the ointment? A) Before arising in the morning B) After breakfast C) After dinner D) At bedtime

Ans: D Feedback: Eye ointments should be instilled when the client can lie back and allow the ointment to dissolve and bathe the eye. This is best accomplished at bedtime.

29. Following cataract removal, discharge instructions will be provided to the client. Which of the following instructions is most important? A) Apply protective patch to both eyes at bedtime. B) Only sleep on back. C) Avoid washing face and eyes for first 24 hours. D) Avoid any activity that can increase intraocular pressure.

Ans: D Feedback: For approximately 1 week, the client should avoid any activity that can cause an increase in intraocular pressure. Clients may sleep on back or unaffected side. Clients may use a clean damp cloth to remove eye discharge and wash face. An eye shield is often ordered for the first 24 hours and during the night to prevent rubbing or trauma to the operative eye.

30. The nurse is working in the triage section of a walk-in clinic. Which triad of common symptoms, when placed together, indicate Ménière's disease? A) Blurred vision, vertigo, nausea B) Syncope, vertigo, ear pain C) Disorientation, vertigo, nausea D) Hearing loss, vertigo, tinnitus

Ans: D Feedback: Hearing loss, vertigo, and tinnitus are common symptoms of many disease processes but, when placed together, indicate Ménière's disease. The other options do not include the accurate triad of symptoms.

2. A nurse is assessing a client for a fracture to the bony orbit. What would the nurse document if her assessment for fracture was positive? A) There is excessive tearing. B) The client's vision is blurred. C) A rust ring is seen around the pupil. D) The client has diplopia.

Ans: D Feedback: If the bony orbit is fractured, the eyes may appear asymmetric, and the client has diplopia or double vision. Excessive tearing, presence of rust rings, or blurry vision does not indicate a fractured bony orbit.

13. At morning report, the nurse learns the assigned client is blind. Which question should the nurse ask the client upon initial assessment? A) "Have you always been blind?" B) "What caused your vision problem?" C) "Are you dependent with your care?" D) "Can you perceive light and motion?"

Ans: D Feedback: Many people who are considered blind perceive light and motion. Establishing this fact can help in developing a plan of care for this client. Establishing cause and length of time for visual impairment is not required for initial care. Asking the client about dependence is important, but the new environment could provide safety issues (even if independent) if no perception of light is identified.

27. The client has been diagnosed with objective vertigo. Which symptom would the nurse relate to the tentative diagnosis? A) Frequency of a headache B) Pain in the outer ear C) Hearing ability fluctuations D) A sensation of things moving

Ans: D Feedback: Objective vertigo includes the sensation that the environment is moving or a sense that things are moving around oneself. The symptoms do not include a headache, pain in the outer ear, and difficulty hearing.

The nurse is working in the emergency department when a physician asks for help as the client is performing a Romberg test. In which position would the nurse stand to be most helpful? A) The nurse would stand directly in front of the client. B) The nurse would stand between the client and physician. C) The nurse would stand across the room but in direct alignment from the client. D) The nurse would stand laterally to the client, opposite side to where the physician is standing.

Ans: D Feedback: The Romberg test is used to evaluate a person's ability to sustain balance. The client stands with the feet together and arms extended. In the event that the client begins to sway (an abnormal result), the nurse is most helpful to stand on the lateral side of the client, opposite side to where the physician is standing to ensure that the client does not fall.

4. You are admitting a client with an acoustic neuroma to your unit. What would you include during the assessment of this client? A) Measure the client's urine output. B) Note the client's height and weight. C) Test the client's ability to sustain balance. D) Test for facial sensation.

Ans: D Feedback: The assessment of a client with an acoustic neuroma includes evaluating hearing function, observing the client's facial movements, and testing for facial sensation. The client's urine output, height and weight, and ability to sustain balance, although important, are not as essential as testing for facial sensation.

The nurse is caring for geriatric clients stating that they are prescribed reading glasses. Some individuals state needing assistance with seeing writing far away, and others need assistance with closer vision. The nurse is correct to understand that the aging visual changes relate to which of the following? A) Changes in refraction B) Changes in the visual field C) Changes in central vision D) Changes in accommodation

Ans: D Feedback: The changes that occur in vision during aging, which include difficulty reading and the need for reading glasses, include changes in accommodation. Accommodation occurs when the ciliary muscles contract or relax to focus an image on the retina.

7. What is located in the cochlea of the inner ear? A) Semicircular canals B) Labyrinth C) Vestibulocochlear nerve D) Organ of Corti

Ans: D Feedback: The fluid motion created by the vibrating stapes excites the nerve endings in the sensitive sound receptors of the organ of Corti located in the cochlea. The labyrinth is the name for the inner ear, and the semicircular canals and vestibulocochlear nerves are other components of the inner ear.

A client states having difficulty noting details on faces or television. Which of the following structures of the eye allows for detailed vision? A) The pupil B) The iris C) The cornea D) The macula lutea

Ans: D Feedback: The macula lutea is composed entirely of cones and allows for detailed vision. It lies in the center of the retina. This client could potentially have macular degeneration. The iris is the highly vascular, pigmented portion of the eye surrounding the pupil that adjusts in response to light. The cornea covers the anterior portion of the eyeball.

14. The nurse is instructing a client on the benefits of a cochlear implant. The client asks, "How am I able to interpret sound?" The nurse credits which of the following as significant in the production of hearing? A) External microphone B) Internal processor C) Amplifier D) Auditory nerve

Ans: D Feedback: The nurse credits stimulation of the auditory nerve in the transmission of the electrical signals to the brain for interpretation. The external processor and internal processor bring the sounds from the environment and send them to the internal processor, which converts to the electrical signal. An amplifier is used with typical hearing aids.

Which of the following nursing diagnosis is most appropriate when caring for a client with deteriorating vision? A) Risk for Injury B) Hopelessness C) Impaired Adjustment D) Altered Sensory Perceptions

Ans: D Feedback: When caring for a client with deteriorating vision, the most appropriate nursing diagnosis focuses on the alteration in sensory perceptions. A "Risk for" diagnosis is appropriate when no current diagnosis is available. There is no data indicating client hopelessness or an impaired adjustment.

19. An elderly client with macular degeneration has received injections of angiogenesis inhibitors. Which assessment finding would indicate the condition is worsening? A) Blurred vision B) Burning sensation of the eyes C) Loss of peripheral field vision D) Central vision impairment

Ans: D Feedback: When the macula becomes irreparably damaged, central vision is lost and the client can only see images via peripheral field. Blurred vision is the initial symptom of the disease and does not signify worsening. Burning sensation is a common adverse reaction to the treatment injection.

You are teaching the daughter how to instill ear drops of her father to remove impacted cerumen. What is important to teach this woman? A. Insert the irrigating syringe deeply. B. Direct the flow of the ear drops toward the eardrum. C. Refrigerate before instillation. D. Place the container in warm water before instillation.

Ans: D If irrigation or instillation of liquids is ordered, the nurse should warm the liquid to body temperature by placing the container in warm water. Cold or hot liquids cause dizziness, and the potential for injury exists if the liquid is hot. The nurse should avoid inserting the irrigating syringe too deeply so as not to close off the auditory canal. The nurse should direct the flow toward the roof of the canal, rather than the eardrum.


Related study sets

Graphing, Measurement and Scientific Method

View Set

Chapter 7: Portable Fire Extinguishers

View Set